krok 2 - Центр тестування при МОЗ Україниkrok 2 stomatology ()...

28
Sample test questions Krok 2 Stomatology ()

Upload: others

Post on 08-Oct-2020

8 views

Category:

Documents


0 download

TRANSCRIPT

Page 1: Krok 2 - Центр тестування при МОЗ УкраїниKrok 2 Stomatology () Терапевтична стоматологiя 2 1. Biopsy material was obtained from the

Sample test questions

Krok 2 Stomatology

()

Page 2: Krok 2 - Центр тестування при МОЗ УкраїниKrok 2 Stomatology () Терапевтична стоматологiя 2 1. Biopsy material was obtained from the

Терапевтична стоматологiя 2

1. Biopsy material was obtainedfrom the focus of a lesion in theretromolar space. Pathohistological findings:cellular polymorphism in the stratumspinosum, increased mitotic number, giantmultinucleate cells, acanthosis, some cellsin the stratum spinosum exhibit signs ofparakeratosis, keratin pearls are detected.These pathohistological findings indicate thefollowing disease:

A. Bowen diseaseB. Lupus erythematosusC. LeukoplakiaD. Lichen ruber planus, verrucous formE. Keratoacanthoma

2. During preventive examination a 40-year-old man presents with the following changes:marginal gingiva is enlarged, torus-shaped,cyanotic, slightly bleeding when touched witha dental probe; there is no pain. Staining thegums with Lugol’s iodine solution results inlight-brown coloring of mucosa. Make thediagnosis:

A. Chronic catarrhal gingivitisB. Acute catarrhal gingivitisC. Exacerbation of chronic catarrhal gingivitisD. Chronic hypertrophic gingivitisE. Generalized periodontitis

3. A patient complains of carious cavity intooth 11. The filling was lost one week ago.The tooth crown is dark, there is residualfilling material at the bottom of the cariouscavity. Vertical percussion is painless. X-rayshows an oval area of bone tissue resorptionwith clear margins, 0.4х0.3 cm in size. The rootcanal is filled by 2/3 of its length. What is themost likely diagnosis?

A. Chronic granulomatous periodontitisB. Chronic fibrous periodontitisC. Chronic granulating periodontitisD. Radicular cystE. Exacerbation of chronic periodontitis

4. A 30-year-old woman complains of mildburning sensation in her lower lip and itsdryness. She peels skin scales off with herteeth. She has been presenting with thiscondition for 10 years. On examination theskin scales are gray and located on the lip fromthe Klein’s line to the center of the vermillionborder from angle to angle of the mouth. Thescales are firmly attached in the center and areloose on the periphery. Their forcible removaldoes not result in erosions. What is the mostlikely diagnosis?

A. Exfoliative cheilitisB. Lupus erythematosusC. Meteorological cheilitisD. Allergic contact cheilitisE. Eczematous cheilitis

5. A 32-year-old patient presents with body

temperature of 38.9oC , general fatigue,impaired speech, inability to eat. Thiscondition has been recurring for the last4 years in autumn and spring. There arevesicles and erosions covered in grayishfibrinous coating on the hyperemic andswollen labial and buccal mucosa. Nikolsky’ssign is negative. What is the most likelydiagnosis?

A. Erythema multiforme exudativumB. Pemphigus vulgarisC. Acute herpetic stomatitisD. Nonacantholytic pemphigusE. -

6. A 36-year-old woman complains of drynessand peeling of her lower lip vermillion border.Both dryness and peeling have been observedfor a month. Application of indifferentointments was ineffective. Objectively thevermillion border of the lower lip is brightred, moderately infiltrated, and denselycovered in white-gray scales, that cannot beremoved without pain and bleeding. Thelesion focus contains concave areas, whileon the periphery there are patches of dullepithelium that look like irregular whitestripes. What is the most likely diagnosis?

A. Lupus erythematosusB. Candidal cheilitisC. Lichen ruber planusD. LeukoplakiaE. Exfoliative cheilitis

7. A 35-year-old woman has complaints ofcosmetic defects of the front upper teethcrowns. The defects have been aggravatingfor the last 10 years. The patient suffersfrom unpleasant sensations when brushingher teeth, and when chemical stimuli areapplied. Objective examination revealeddefects localized in the enamel of the frontupper teeth vestibular surface. The defects areoval, saucer-shaped, and have clear margins.Response to probing and cold stimuli waspositive. Make the diagnosis:

A. Enamel erosionB. Enamel hypoplasiaC. Cuneiform defectD. Chemical necrosis of the toothE. Hyperesthesia of tooth hard tissues

8. A 40-year-old man had his root canal ofthe 34 tooth filled due to chronic fibrousperiodontitis. Soon the treated place becamepainful. On X-ray the root canal of the 34tooth is filled to the root apex. What tacticsshould the dentist choose to manage the pain?

A. To prescribe physiotherapeutic proceduresB. To rinse with antiseptic mouthwashC. To make insicion along the mucogingivalfoldD. To provide conduction anesthesiaE. To provide infiltration anesthesia

Page 3: Krok 2 - Центр тестування при МОЗ УкраїниKrok 2 Stomatology () Терапевтична стоматологiя 2 1. Biopsy material was obtained from the

Терапевтична стоматологiя 3

9. A 20-year-old man complains ofspontaneous pain attacks (5-7 minutes) in thearea of 36 that occur for the last 24 hours.Objectively in 36 there is a deep carious cavityon the masticatory surface. Probing is painfulat one point, response to cold stimulus ispainful, with an aftereffect that lasts for 5minutes. Percussion is painless. X-ray showsroot canals to be slightly curved, canal lumenis clearly visible. The tooth is planned to beused as an abutment for a dental bridge. Whatis the optimal treatment method in this case?

A. Vital extirpationB. Biological approachC. Vital amputationD. Devital amputationE. Filling of the carious cavity

10. A 34-year-old man complains of acutespasmodic pain in the region of his upper jawon the left that is aggravating when affectedby cold stimuli. Toothache irradiates to theear and temple. He had acute toothache inthe 27 tooth one year ago, but he did notconsult a dentist. Pain recurred three days ago.Objectively: the 27 tooth has a carious cavitycommunicating with the dental cavity. Probingof the opened carious cavity is extremelypainful. X-ray picture shows widening ofperiodontal fissure at the root apex of the 27tooth. What is the most likely diagnosis?

A. Exacerbation of chronic fibrous pulpitisB. Exacerbation of chronic granulatingperiodontitisC. Exacerbation of chronic fibrousperiodontitisD. Acute diffuse pulpitisE. Exacerbation of chronic gangrenous pulpitis

11. A 38-year-old woman complains ofburning pain in her lips and angles of hermouth, their dryness. She has an 8-year-long history of diabetes mellitus. Objectively:the vermillion border is dry, congestivelyhyperemic, covered in scales of variablesize. In the angles of the mouth there arefissures covered in white coating, the skinis macerated. What ointment should beprescribed for topical treatment in the givencase?

A. ClotrimazolB. InterferonC. PrednisoloneD. LanolinE. Erythromycin

12. A 23-year-old man complains of gumbleeding when he brushes his teeth or eatssolid food. Objectively: the gums of thefront lower jaw are hyperemic, swollen andbleeding when palpated. Oral and gingivalmucosa in other areas are not affected. Theocclusion is deep. The teeth are firm, exceptfor the 41 and 31 (degree 1 mobility). X-rayshows resorption of the alveolar septum in

the area of the 41, 42, 32, and 31 teeth up to1/3 of the root length. What is the most likelydiagnosis?

A. Localized periodontitisB. Generalized periodontitis, initial stageC. Generalized periodontitis, stage ID. Catarrhal gingivitisE. Parodontosis, stage I

13. A 45-year-old man complains of persistingdull pain in 46 that occurs in response tothermal and chemical stimuli. Examinationof 46 detected in the precervical area of itsvestibular surace a deep carious cavity filledwith soft pigmented dentin. Probing is sharplypainful at one point. Cold water stimulusinduces a dull pain that gradually intensifies.Make the diagnosis:

A. Chronic fibrous pulpitisB. Acute deep cariesC. Chronic deep cariesD. Chronic gangrenous pulpitisE. Chronic fibrous periodontitis

14. A 38-year-old man came to the dentist.After history-taking and examination hewas found to have exacerbated generalizedperiodontitis of the II degree with periodontalpockets 3-5 mm deep. After the localfactors were removed and anti-inflammatorytreatment was provided to the patient, it isnecessary to remove the periodontal pockets.What method should be applied in this case?

A. CurettageB. GingivectomyC. GingivotomyD. Flap surgeryE. Vestibuloplasty

15. A 38-year-old man, an arc welder, is achain smoker. He came to the dentist to havea denture made for him. Howerver, in themiddle of his lower lip against the backgroundof unchanged vermillion border there is agray-white polygonal lesion 6x4 mm with clearmargins. The lesion is covered with tightlyattached small scales and is level with thevermillion border (neither protruding norretracted). Palpation of the lesion focus ispainless, the lesion has no thickened base;lateral palpation detects thickened surfaceof the lesion. Regional lymph nodes areunchanged. Make the provisional diagnosis:

A. Localized precancerous hyperkeratosisB. Verrucous precancerC. Lupus erythematosusD. Cancer of the lower lipE. Lichen ruber planus, hyperkeratotic form

16. A 53-year-old patient complains of anulcer on the lateral surface of the tongue. Theulcer appeared 6 months ago in the resultof a trauma caused by sharp tip of the 37tooth metal crown. A dentist replaced the

Page 4: Krok 2 - Центр тестування при МОЗ УкраїниKrok 2 Stomatology () Терапевтична стоматологiя 2 1. Biopsy material was obtained from the

Терапевтична стоматологiя 4

crown with the one of better quality andprescribed keratoplastic drugs. Despite thesemeasures the ulcer continues to grow. Latelythere has been pain during talking, chewing,swallowing; sometimes the pain irradiatesto the pharynx. Objectively: on the lateralsurface of the tongue there is a painful ulcerwith uneven raised dense margins and lumpyfloor covered with grayish necrotic coating.What diagnosis is most likely?

A. Cancer of the tongue lateral surfaceB. Trophic ulcerC. Traumatic ulcerD. Vincent’s necrotizing ulcerative stomatitisE. Tuberculous ulcer

17. A 20-year-old young woman complainsof pain and marked bleeding in her gumswhen she brushes her teeth and eats evensoft food. Objectively her gingival mucosa isswollen, hyperemic and bleeds even on theslightest touch. The teeth are covered withmoderate amount of soft dental plaque. Whattoothpaste should be recommended to thispatient as a part of complex treatment of hercondition?

A. Herbal toothpasteB. Mineral-rich toothpasteC. Fluoride toothpasteD. Antifungal toothpasteE. Gel toothpaste with microelements

18. A 50-year-old man complains of gumdiscoloration, pain when eating spicy food,weakness, and irritability. He notes thatperiodically spots and vesicles appear on hisskin. For a long time he has been workingin the galvanic workshop of a factory. Onexamination the patient is pale, he has agray-black border on his gums, and on hisoral mucosa there are single erosions that arepainful on palpation. What is the most likelyprovisional diagnosis?

A. Lead salt poisoningB. Necrotizing ulcerative stomatitis (Vincentstomatitis)C. Mercury salt poisoningD. Acute leukemiaE. Erythema multiforme exudativum

19. A 32-year-old man was diagnosed withulcerative gingivitis. After processing thepatient’s oral cavity with an antiseptic, thedoctor applied anesthesia and removed dentalplaque and necrotic coating. Final part of themedical appointment is the application ofgingival dressing. What etiotropic drug shouldthis dressing contain?

A. MetronidazoleB. Contrykal (Aprotinin)C. MethyluracilD. Pimafucin (Natamycin)E. Terrilytin

20. A 28-year-old patient complains of painand bleeding of gums in the frontal part of theupper jaw on the left. Two years ago, the 22tooth was covered with a porcelain-fused-to-metal crown. Objectively: interdental papillabetween the 21 and 22 tooth is hypertrophied,markedly hyperemic, overlaps the crown ofthe 22 by 1/3 of its height, bleeds whentouched. Periodontal pocket between the 21and 22 tooth is 4 mm deep. Artificial crown isplaced on the gingival margin. X-ray revealsresorption of the interalveolar septa betweenthe 21 and 22 tooth by 1/3 of their height.Specify a priority action in the treatment ofthis patient:

A. Removal of the artificial crownB. GingivectomyC. Anti-inflammatory therapyD. Curettage of the periodontal pocketE. Sclerotherapy

21. A 45-year-old patient complains ofbleeding gums and bad breath. Afterexamination the patient was provisionallydiagnosed with chronic generalizedperiodontitis of the II degree. To make thefinal diagnosis, it is necessary to:

A. Perform an X-rayB. Measure the depth of the periodontalpocketsC. Determine teeth mobilityD. Perform Kulazhenko test (capillaryresistance test)E. Perform Kotzshke test (periodontal pocketsuppuration test)

22. A 67-year-old woman complains of anulcer on her oral mucosa and pain wheneating. She has a history of decompensatedvalvular heart diseases. Examination detectedin the posterior portion of her left buccalregion an irregularly shaped ulcer with white-gray coating. Surrounding mucosa is notinflamed. Make the provisional diagnosis:

A. Trophic ulcerB. Necrotizing ulcerative stomatitisC. Cancerous ulcerD. Decubitus ulcerE. Tuberculous ulcer

23. A 53-year-old man complains ofincreased teeth sensitivity to chemical stimuli.Objectively: the gums are pale pink, roots areexposed by 1/3 of their length. Small amountof dental deposit is observed. The 15, 14, and24 present with cuneiform defects. Probing ofthe exposed cervices and defects is painful.What is the most likely diagnosis?

A. Periodontosis, I degreeB. Catarrhal gingivitisC. Periodontitis, II degreeD. Periodontitis, I degreeE. Ulcerative gingivitis

Page 5: Krok 2 - Центр тестування при МОЗ УкраїниKrok 2 Stomatology () Терапевтична стоматологiя 2 1. Biopsy material was obtained from the

Терапевтична стоматологiя 5

24. An 18-year-old young man complains ofa cavity in 27 and pain when eating solidfood. He has a history of severe pain in 27one year ago; he requested no medical helpat the time. Objectively on the masticatorysurface of 27 there is a deep carious cavitythat communicates with the pulp chamber.The cavity is nearly completely filled withovergrown granulation tissue. Probing ofthe granulations leads to mild pain andbleeding. Percussion is painless. EPT is 45microamperes. Make the diagnosis:

A. Chronic hypertrophic pulpitisB. Chronic fibrous pulpitisC. Chronic hypertrophic gingivitisD. Chronic fibrous periodontitisE. Chronic granulating periodontitis

25. A 25-year-old man complains of a cavityin his lower left tooth, pain after eating sweet,sour, or solid food. Objectively in tooth 16there is a deep carious cavity filled with light-colored softened dentin. Probing of the cavityfloor is painful, pain stimulus is painful, butcold abates immediately after the stimulus isremoved. What is the most likely diagnosis?

A. Acute deep cariesB. Chronic deep cariesC. Acute focal pulpitisD. Chronic fibrous pulpitisE. Acute median caries

26. A 50-year-old man complains of painfuldefect that manifests as a crack in his lowerlip. The defect appeared one month ago.Objectively the lower lip is dry, intactnessof the vermillion border of the lower lip islinearly disturbed by a lesion that is located inthe middle of the lip, is covered in dried blood,and extends to the labial mucosa. What is themost likely diagnosis?

A. Chronic lip fissureB. Atopic cheilitisC. Chronic lymphedema of lipsD. Miescher cheilitis granulomatosaE. Melkersson-Rosenthal syndrome

27. A group of experts conducts epidemiologicinvestigation of certain age groups by severityof signs of periodontal damage and their needfor treatment. What index allows to studythese criteria?

A. CPITN (WHO)B. PMA (Parma)C. PDI (Ramfjord)D. OHI-S (Green-Vermillion)E. PI (Russel)

28. During the treatment of acute deep cariesin 16 in an 18-year-old girl, the pulp chamberwas accidentally perforated. Perforation ispunctate and located in the area of a pulphorn. What treatment tactics would beadvisable in this case?

A. Biological approachB. Vital extirpationC. Devital amputationD. Devital extirpationE. Insulating inlay and permanent filling

29. A 30-year-old woman addressed a dentalsurgeon with complaints of uncomfortablesensation of pressure in her upper right tooth,which aggravates due to hot stimulus, and foulsmell from the mouth. Objectively: there isa deep carious cavity in the 17 tooth, whichpenetrates into the tooth cavity. Deep probingcauses severe pain, percussion of the 17 toothis painful. X-ray: there is slight widening ofthe periodontal fissure near the root apex.Electric pulp test - 70 microamperes. Whatfinal diagnosis can be made?

A. Chronic gangrenous pulpitisB. Chronic fibrous pulpitisC. Acute purulent pulpitisD. Chronic fibrous periodontitisE. Exacerbation of chronic fibrousperiodontitis

30. A woman complains of pain responseto thermal stimuli in her lower right jaw.The pain quickly abates after the stimulus iswithdrawn. Objectively on the masticatorysurface of the 26 there is a carious cavity withnarrow ”entrance” located within the mantledentin. The dentin is soft and light-colored.Percussion of the 26 is painless. Make thediagnosis:

A. Acute median cariesB. Chronic superficial cariesC. Acute deep cariesD. Chronic median cariesE. Chronic deep caries

31. A 24-year-old patient came to the dentistcomplaining of chalky lesions on the frontteeth. Objectively teeth 11, 21, 31, 41 presentwith chalky lesions separated by areas ofhealthy unchanged enamel. Lesion surface iscoarse; there is no response to thermal stimuli.Childhood years of the patient were spentin the area with fluorine level of 1.8 mg/L indrinking water. Make the diagnosis:

A. Endemic fluorosisB. Enamel hypoplasiaC. Enamel hyperplasiaD. Acute superficial cariesE. Chronic superficial caries

32. A 48-year-old patient has come tothe hospital with complaints of defects inthe paragingival area and slight sensitivityto thermal stimuli. Objectively: there arehard tissue defects that resemble a wedgewith smooth polished walls on precervicalvestibular surface of the 23 and 24 teeth.Thermal test is slightly positive. What is themost likely diagnosis?

Page 6: Krok 2 - Центр тестування при МОЗ УкраїниKrok 2 Stomatology () Терапевтична стоматологiя 2 1. Biopsy material was obtained from the

Терапевтична стоматологiя 6

A. Cuneiform defectB. Enamel necrosisC. Acute deep cariesD. Enamel erosionE. Endemic fluorosis

33. A 34-year-old man presents with persistingdull pain in his tooth, which aggravates onbiting. One week ago the tooth was treated fordeep caries. Objectively on the masticatorysurface of tooth 36 there is a filling, percussionis painful, there is a supracontact observedin the area of 36. X-ray shows unchangedperiodontium. What mistake was made whentooth 36 was being filled?

A. High fillingB. Filling without insulation layerC. Gingival attachment is disturbedD. Insulation layer exceeds borders of thedentinoenamel junctionE. Filling without medicinal substance sealedinside

34. A 35-year-old man complains of persistingpain in tooth 24, which intensifies on biting.Objectively on the distal masticatory surfaceof tooth 24 there is a deep carious cavityfilled with food debris. Percussion of cavitybottom is painless, there is no pain responseto thermal stimuli. Percussion of tooth 24 isacutely painful. X-ray shows no pathologicchanges of periapical tissues in the area ofroot apices of 24. What is the most likelydiagnosis?

A. Acute serous periodontitisB. Acute suppurative periodontitisC. Acute suppurative pulpitisD. Acute diffuse pulpitisE. Exacerbation of chronic periodontitis

35. A 38-year-old man complains of sensationof a foreign body on his tongue anddevelopment of gag reflex during talking.The signs appeared after the prolongedtaking of antibiotics. Objective examinationdetected thickened and pigmented filiformpapillae enlarged to 2-3 cm in size.Histologically papillar hyperplasia andmarked keratinization without changes in thesurrounding tissues were detected. What isthe most likely diagnosis?

A. Black hairy tongueB. Median rhomboid glossitisC. Fissured tongueD. Glossitis areata exfoliativaE. Geographic tongue

36. A new dental polyclinic is being openedin the city. There are to be 38 doctorsin the dental surgery department. Howmany department heads should supervisethis number of personnel according to thenormatives?

A. 3B. 4C. 2D. 1E. 5

37. A 35-year-old patient undergoes treatmentof chronic fibrous pulpitis of tooth 25.The canal is to be filled using the warmgutta-percha vertical condensation technique.What instrument is needed for gutta-perchacondensation?

A. PluggerB. SpreaderC. Endodontic probeD. Root needleE. K-file

38. A 27-year-old woman complains of anaesthetical defect in the area of her uppercentral incisors. She was diagnosed withchronic deep caries, Black class IV. Aestheticrestoration of teeth 11 and 21 is planned.What material has optimal combination ofstrength and aesthetical properties?

A. Hybrid compositesB. Macrofilled compositesC. Microfilled compositesD. CompomersE. Liquid composites

39. A 30-year-old man complains of lostdental filling on his upper right jaw.Objectively in tooth 16 there is a deep cariouscavity filled with dense pigmented dentin.What dentin layer forms as the result of toothirritation in the course of caries process?

A. TertiaryB. SecondaryC. PrimaryD. HybridE. Predentin

40. A 32-year-old man was diagnosed withchronic fibrous pulpitis of tooth 27. Vitalextirpation is chosen as the treatment method.Amide anesthetic is to be used for tuberal andpalatal anesthesia. What anesthetic solutionneeds to be administered in this case?

A. 3% mepivastesin (mepivacaine)B. 5% anaesthesin (benzocaine)C. 10% lidocaineD. 2% novocaine (procaine)E. 2% dicain (tetracaine)

41. During preventive examination, a 20-year-old man was found to have enamel defectsthat look like irregularly shaped coarse whitespots in the vestibular precervical area ofteeth 11 and 12. The spots stain with 2%methylene blue solution. Make the diagnosis:

Page 7: Krok 2 - Центр тестування при МОЗ УкраїниKrok 2 Stomatology () Терапевтична стоматологiя 2 1. Biopsy material was obtained from the

Терапевтична стоматологiя 7

A. Acute initial caries of teeth 11 and 12, Blackclass VB. Acute superficial caries of teeth 11 and 12,Black class VC. Acute initial caries of teeth 11 and 12, Blackclass IIID. Acute superficial caries of teeth 11 and 12,Black class IIIE. Focal enamel hypoplasia of teeth 11 and 12

42. An 18-year-old young man complains oftooth sensitivity in his lower left jaw wheneating sweet food. Examination shows enameldefects in the fissures on the masticatorysurface of tooth 37. The enamel there is dullwhite and fragile when (chips off) probed.Make the diagnosis:

A. Acute superficial caries of tooth 37B. Chronic superficial caries of tooth 37C. Acute initial caries of tooth 37D. Chronic initial caries of tooth 37E. Chronic median caries of tooth 37

43. A 28-year-old woman came to the dentistfor oral cavity sanation. She was diagnosedwith acute deep caries of tooth 24. She hasa history of seasonal allergic rhinitis anddrug allergy that manifests as a skin rashand Quincke edema. She does not rememberwhether she has been given local anesthesiaat the dentist’s office previously. What tacticsshould the dentist choose regarding theanesthesia in this case?

A. Referral to the allergologist for consultationand anesthetic allergy testingB. Perform a cutaneous anesthetic allergy testin the dental chair immediately before givinganesthesiaC. Perform an anesthetic scratch test inthe dental chair immediately before givinganesthesiaD. Use amide local anestheticE. Use ether anesthetic

44. A 20-year-old pregnant woman withthe term of 22 weeks complains of thermalsensitivity of her upper front teeth. Thesensitivity developed one month ago. She wasdiagnosed with acute initial caries of teeth 12,11, 21, 22, Black class V. Fedorov-Volodkinahygiene index is 1.8. What treatment tactics

would be optimal in this case?

A. Professional teeth cleaning andremineralization therapyB. Professional teeth cleaning and filling ofteeth 12, 11, 21, and 22C. Postpone the treatment until 30 weeks ofgestationD. Remineralization therapy and filling ofteeth 12, 11, 21, and 22E. Professional teeth cleaning,remineralization therapy, and filling of teeth12, 11, 21, and 22

45. After extraction of tooth 46, a 41-year-old man noticed a dark defect on the distalcontact surface of tooth 45. The defect islimited to the enamel, has blurred marginsand dark brown color. The affected tissuecrumbles when probed. Make the diagnosis:

A. Chronic superficial caries, Black class IIB. Chronic initial caries, Black class IVC. Chronic median caries, Black class IVD. Chronic initial caries, Black class IIE. Chronic superficial caries, Black class IV

46. A 25-year-old man was diagnosed withacute deep caries of tooth 13, Black class V.Photopolymer composite material is plannedto be used for aesthetical restoration of tooth13. What material should be used as a liningfor the floor of the carious cavity in this case?

A. Calcium-containing pasteB. Dentin pasteC. Devitalizing pasteD. Resorcin-formalin pasteE. Iodoform paste

47. A patient needs endodontic treatment oftooth 21. The canal is being processed withmanually operated endodontic instruments.Name one such instrument made by meansof conical spiral threading (turning) of a steelwire with round cross-section (milling):

A. H-fileB. K-fileC. K-reamerD. SpreaderE. Plugger

Page 8: Krok 2 - Центр тестування при МОЗ УкраїниKrok 2 Stomatology () Терапевтична стоматологiя 2 1. Biopsy material was obtained from the

Хiрургiчна стоматологiя 8

1. A 66-year-old woman complains of dullpain in her left parotid-masseteric regionand increasing facial assymetry that she firstnoticed 2-3 months ago. Objectively thepatient is undernourished, pale, and suffersfrom the left-sided paresis of the mimicmuscles. In front of her earlobe, there isa lumpy infiltration without clear margins.The infiltration is 4x5 cm in size. It is fusedto the surrounding tissues and moderatelypainful. No saliva could be produced from theduct of the left parotid gland. Submandibularand cervical lymph nodes on the left areenlarged. Survey X-ray of the mandible showsno changes in the structure of the bone tissue.What is the most likely diagnosis?

A. AdenocarcinomaB. Actinomycosis of the left parotid-massetericregionC. Herzenberg pseudoparotitisD. Mixed tumor of the left parotid glandE. Exacerbated chronic parotitis

2. After extraction of tooth 27 the dentalsurgeon suspected that the maxillarysinus was perforated. To clarify thisprovisional diagnosis the doctor desided toconduct an oronasal communication test(Valsalva maneuver). What manipulations arenecessary for this test?

A. Pinch the nostrils together to occlude thepatient’s nose and ask the patient to blowthrough the nose, while observing the socketof 27B. Pinch the nostrils together to occlude thepatient’s nose and ask the patient to blowthrough the mouth, while observing the socketof 27C. Ask the patient to blow through the nose,while observing the socket of 27D. Ask the patient to inhale through the nose,pinch the nostrils together, and exhale throughthe mouthE. Ask the patient to blow through the mouth,while observing the socket of 27

3. The 15 tooth must be extracted. The toothcrown is retained. What instrument should beused in this case?

A. Forceps with S-shaped handlesB. Straight forcepsC. Bayonet forcepsD. Left-sided forceps with S-shaped handlesE. Right-sided forceps with S-shaped handles

4. A 48-year-old man complains of atumor-like formation on the hard palate.Examination detected a semi-sphericalprotrusion with clear margins in the anteriorportion of the palate. Teeth 11 and 21 areintact. X-ray shows a homogeneous focusof bone tissue rarefaction. The focus is3.5x2.5 cm and has clear margins. Againstthe background of rarefied bone tissuethere are projections of intact teeth 11 and

21; periodontal fissure is visible. Make theprovisional diagnosis:

A. Nasopalatine duct cystB. Follicular cyst of the maxillaC. Radicular cyst of the maxillaD. Maxillary ameloblastomaE. Giant-cell tumor of the maxilla

5. A 28-year-old man complains of pain in theinfraorbital and parotid region on the left. Onexamination: hemorrhage occurs in the lowereyelid and conjunctiva of the left eye, thereare signs of crepitation and step deformity ofthe eyesocket lower edge. The mouth opensby 1 cm. Make the diagnosis:

A. Zygomatic bone fractureB. Malar arch fractureC. Left articular process fractureD. Traumatic arthritis of thetemporomandibular jointE. Hematoma of the infraorbital region

6. A patient received a chemical burn to theface. The burn was caused by sulfuric acid.What substance should be used in the woundprocessing to neutralize this acid?

A. AlkaliB. Flowing waterC. AntisepticD. AcidE. Alcohol

7. A 50-year-old man was diagnosed withrecurrent sialolithiasis with the sialolithlocated deep within the submandibularsalivary gland. Choose the optimal treatmenttactics:

A. Submandibular gland excisionB. Radiation therapyC. SclerotherapyD. Removal of the sialolith while retaining theglandE. Conservative pharmacotherapy

8. An 18-year-old girl needs consultationof the dentist regarding a neoplasm thatappeared in the frontal area of her face.On the frontal area there is a pink-redsemicircular spot on her skin 3-4 cm in size,with clear margins. When pressed, the skin inthe affected area discolors and becomes pale.As soon as the pressing stops, the skin againassumes its pink-red color. When the patientbows her head, the spot darkens and enlargesin volume; when the patient lifts her head, thespot gradually returns to its initial pink-redcolor. Make the diagnosis:

A. Capillary hemangiomaB. Cavernous hemangiomaC. Birthmark (nevus)D. LymphangiomaE. Neurofibromatosis

9. A 30-year-old patient is diagnosed with

Page 9: Krok 2 - Центр тестування при МОЗ УкраїниKrok 2 Stomatology () Терапевтична стоматологiя 2 1. Biopsy material was obtained from the

Хiрургiчна стоматологiя 9

acute suppurative odontogenic periostitis ofthe upper left jaw originating from tooth 23.The crown of 23 on the left is destroyed withcaries by 1/3. Teeth 22 and 24 are intact. Spot-film X-ray shows widening of the periodontalfissure of 23. What treatment would be themost advisable in this case?

A. Periosteotomy and pharmacotherapyfollowed by treatment of the causative toothB. Extraction of the causative tooth,periosteotomyC. Extraction of the causative tooth,physiotherapyD. Extraction of the causative tooth,pharmacotherapyE. -

10. A 45-year-old man presents with facialasymmetry due to a dense isolated infiltrationin his right buccal area; the skin over theinfiltration is cyanotic, thinned out; in thecenter of the infiltration there is a fistula. Inthe oral cavity the crown of 46 is destroyedby 2/3, along the mucogingival fold the bandconnecting the tooth with the fistula can bepalpated. Make the diagnosis:

A. Migrating facial granulomaB. Chronic mandibular osteomyelitisC. Cheek furuncleD. Odontogenic lymphadenitisE. Actinomycosis

11. A 28-year-old man has received a hitto the chin. He complains of pain in theright side of his face. Examination showsasymmetrical face due to tissue swelling inthe upper right parotid area, mouth openingis painful, limited, the jaw is displaced to theright, right-sided crossbite is observed. Makethe provisional diagnosis:

A. Displaced traumatic fracture of the cervixof the mandibular articular processB. Displaced traumatic unilateral fracture ofthe body of the mandibleC. Displaced traumatic double fracture of thebody of the mandibleD. Displaced traumatic mental fracture of themandibleE. Displaced traumatic central fracture of themandible

12. A 35-year-old man has been hospitalizedinto a maxillofacial unit with complaints ofmobility of the 38, 37, and 36 teeth and afistulous tract in the socket of the extracted 35tooth. The condition has been persisting forover 3 months. Insertion of a grooved probeinto the fistulous tract palpated exposedcoarse bone fragment that easily moved underpressure. X-ray of the lower jaw demonstratesa focus of bone tissue destruction, with a spotof dense bone tissue 0.5х0.3 cm in size. Makethe diagnosis:

A. Chronic osteomyelitisB. Acute osteomyelitisC. Exacerbation of chronic osteomyelitisD. Chronic periostitisE. Actinomycosis

13. A 29-year-old man came to the surgerydepartment of a dental hospital. Hecomplains of painful facial swelling on theright that appeared 1.5-2 months ago andhas been gradually increasing since then.Objectively mouth opening is unrestricted,the face is asymmetrical due to a swelling inthe right parotid-masseteric region. The skinover the edema is hyperemic and swollen;palpation detects a painful infiltration 3 cmin size. Massage in this region results indischarge of clear saliva from the excretoryduct of the right parotid salivary gland. Whatdisease can be suspected in this case?

A. Chronic suppurative lymphadenitis of theright parotid-masseteric regionB. Acute suppurative parotitis on the rightC. Furuncle of the right parotid-massetericregionD. Chronic interstitial parotitis on the rightE. Submasseteric abscess on the right

14. A 25-year-old woman made anappointment with the dental surgeon fororal cavity sanation. Objectively the crownof tooth 37 is destroyed by 2/3. Gingivalmucosa around tooth 37 is without changes.What anesthesia should the dental surgeongive to the patient for the procedure of toothextraction?

A. Mandibular and buccal anesthesiaB. Intraoral infraorbital nerve blockC. Tuberal anesthesiaD. Mandibular anesthesiaE. Mental nerve block

15. A 28-year-old man came to themaxillofacial surgery department. Hecomplains of a cutaneous neoplasm in hisright submandibular region. The neoplasmappeared approximately 2 years ago. Inthe process of shaving, the neoplasmwas frequently injured. He was diagnosedwith papilloma. In the neoplastic area,examination shows an ulcer against thebackground of hyperemic skin. What tacticsshould the dental surgeon choose?

A. Prevention of malignant transformation ofthe papilloma. Removal of the papillomaB. Antibacterial and anti-inflammatorytherapy, observationC. Removal of the papilloma and postoperativeradiation therapyD. Anti-inflammatory therapy followed byremoval of the papillomaE. Anti-inflammatory therapy followed byobservation

16. The dental surgery department received a

Page 10: Krok 2 - Центр тестування при МОЗ УкраїниKrok 2 Stomatology () Терапевтична стоматологiя 2 1. Biopsy material was obtained from the

Хiрургiчна стоматологiя 10

women with complaints of swollen right sideof the fase and indisposition. According toher medical history, one week ago she senseditching in the right side of her face and noticedthere a small dense nodule. She startedto apply various ointments. Swelling wasgradually increasing, the woman developedpain and fever. Objectively her conditionis of moderate severity, body temperatureis 39oC . Her right buccal region is swollen,with hyperemic skin; in the center there is anoval edema elevated above the surroundingtissues with a black scab visible on its apex.Palpation detects a painful infiltration, 3.5cm in diameter, and enlarged and painfulsubmandibular lymph nodes. Mouth openingis slightly impaired, oral cavity is clean. Makethe diagnosis:

A. FuruncleB. CarbuncleC. Phlebitis of the facial veinD. Acute lymphadenitisE. Acute lymphangitis

17. A 38-year-old man after a domesticaccident complains of pain and mobilityof his upper teeth, problems with eating.Objectively: soft tissues edema. The 11 and 21teeth are displaced towards the palate, mobile(II degree), painful on percussion. Mucosasurrounding the affected teeth is hyperemicand swollen. X-ray demonstrates widenedperiodontal fissure of the 11 and 21. Choosethe treatment method:

A. Setting of the teeth and their fixation with aflat occlusal splintB. Extraction of the 11 and 21 teethC. Removal of tooth pulp in the 11 and 21teethD. Immobilization with mouthguardE. -

18. A patient was diagnosed with cancer of theright parotid gland, T2N2M0. What treatmentmethod should be chosen in this case?

A. Combined therapyB. Radiation therapyC. ChemotherapyD. Surgical removal of the neoplasmE. Surgical removal of the lymph node

19. A 27-year-old woman complains of generalweakness and pain in her lower jaw duringswallowing. Two days ago she developed painin tooth 38. Mouth opening is impossible.What anesthesia should be given to thepatient before examination of the oral cavityand extraction of 38?

A. Berchet-DubovB. MentalC. VerlotskyD. TuberalE. Infiltration

20. A man complains of destroyed crown ofthe 16 tooth. Objectively the crown portionof the 16 tooth is completely destroyed. Thepatient is diagnosed with chronic granulatingperiodontitis of the 16 tooth and this toothneeds to be extracted. What type of anesthesiashould be used for this procedure?

A. Tuberal and palatal anesthesiaB. Infraorbital anesthesiaC. Tuberal anesthesiaD. Infraorbital and tuberal anesthesiaE. Tuberal and incisor anesthesia

21. During application of tuberal anaesthesiathe patient developed rapidly increasingtissue edema and reduced mouth opening.What resulted in such a condition?

A. Vascular traumaB. Muscle trauma during anaesthesiaapplicationC. Nerve trunk traumaD. Intolerance to the anaestheticE. Anaphylactic shock

22. A 38-year-old man needs to have his12 tooth extracted. What type of anesthesiashould be used in this case?

A. Infraorbital and incisorB. Infraorbital and palatalC. IncisorD. Infraorbital, palatal and incisorE. Infraorbital

23. A woman came to the dental surgeonwith complaints of teeth mobility. Afterobjective examination and X-ray analysis shewas diagnosed with generalized periodontitisof the 1-2 degrees of severity. Which teethof those affected by periodontitis should beextracted?

A. With degrees 2-3 of tooth mobilityB. Intact teethC. Teeth with painful percussionD. With degree 1 of tooth mobilityE. Carious teeth

24. After the inflammatory process in theparotid area a woman developed frequentpain attacks resembling electric current in herface on the right. The attacks last for 15-20minutes. The most likely diagnosis is:

A. Trigeminal neuralgiaB. Tympanic plexus neuralgiaC. Trigeminal neuritisD. Exacerbation of chronic maxillary sinusitisE. Exacerbation of chronic osteomyelitis

25. A woman complains of a neoplasm in hersublingual area. The neoplasm makes eatingand speaking difficult. In the sublingual areaexamination detects a soft-elastic neoplasm1cm in diameter with smooth surface andclear margins. The sign of fluctuation isnoticeable on palpation. Oral mucosa is

Page 11: Krok 2 - Центр тестування при МОЗ УкраїниKrok 2 Stomatology () Терапевтична стоматологiя 2 1. Biopsy material was obtained from the

Хiрургiчна стоматологiя 11

stretched tightly over the neoplasm and isblue-tinged and semi-transparent. What is themost likely diagnosis?

A. Ranula of the sublingual glandB. AdenolymphomaC. Mixed tumor of the submandibular glandD. Hemangioma of the submandibular regionE. Calculous sialoadenitis

26. An 18-year-old girl in her childhoodunderwent a surgery for complete bilateralcleft upper lip. Examination detects multiplescars on the markedly flat upper lip. Thereis no visible asymmetry, but the upper lip isslightly deformed. Wide bases of the wingsof her nose are symmetrically displacedlaterally and posteriorly. Her nasal septumis shortened. The tip of the nose is bifurcatedand drawn to the upper lip. Her nasal dorsumis arcuate. What changes occurred in thepatient’s jaws?

A. The upper jaw is underdeveloped, theintermaxilla is frontally displaced due tointerrupted labial muscle layerB. The lower jaw is underdeveloped due toreduced masticatory load, the upper jaw isflattenedC. The upper jaw is underdeveloped in itsfrontal area, while its lateral areas remainwithout pathologic changesD. Both upper and lower jaws areunderdeveloped due to reduced masticatoryload and flattened in their frontal areasE. Both upper and lower jaws are flattenedin their lateral areas due to disturbed nasalbreathig

27. A man complains of pain in his lower rightjaw and impaired mouth opening. Objectivelyon visual examination his face is symmetrical,mouth opening is slightly impaired, trismusof the 1 degree is observed. Mucosa behindtooth 47 is swollen, hyperemic, and painful onpalpation. Medial cusps of tooth 48 are visible,while the tooth itself has not fully erupted.What provisional diagnosis can be made?

A. PericoronitisB. Exacerbation of chronic granulatingperiodontitisC. Acute odontogenic osteomyelitisD. Acute odontogenic periostitisE. Abscess of the pterygomandibular space

28. A 37-year-old woman came to the dentistto have her 25 tooth extracted. What type ofanesthesia should be used in this case?

A. Unilateral tuberal, infraorbital, and palatalanesthesiaB. Unilateral infraorbital and palatalanesthesiaC. Unilateral tuberal and palatal anesthesiaD. Unilateral infraorbital and incisoranesthesiaE. Unilateral central anesthesia

29. A 2-year-old child received a dentaltrauma. Objectively the crowns of 51 and61 are shorter than the crowns of adjacentteeth by 1/3. Mucosa in the area of 51 and61 is hyperemic and swollen. X-ray shows noperiodontal fissure in the apical area of theroots of 51 and 61. What treatment tacticswould be optimal in this case?

A. Dispensary observationB. Extraction of teeth 51, 61C. Reposition of teeth 51, 61D. Ligature splintingE. Reimplantation

30. A 48-year-old woman complains oflow-grade fever and a gradually enlargingulcer on her gingival mucosa near themolars; the teeth in the affected area aremobile. Objectively on the gingival mucosabetween the lower left molars there aretwo superficial sharply painful ulcers withundermining margins. Floor of the ulcersis granulated and covered in yellow-graycoating. Small tubercles surround the ulcers.Tooth cervices are exposed, pathologic toothmobility is observed. Regional lymph nodesare enlarged, painful, and matted togetherinto dense clusters. Make the provisionaldiagnosis:

A. TuberculosisB. SyphilisC. Acute aphthous stomatitisD. Cancerous ulcerE. Trophic ulcer

31. A 36-year-old man complains of an ulceron his lower left lip. Three weeks ago a smalround red spot appeared on his lip. Eventuallyit became more dense and noticeable and 2-3 days ago developed an ulcer in its center,while under the lower jaw appeared multiplepainless nodules. Objectively on the mucosaof the lower left lip there is an oval ulcer,1.2 cm in diameter, with smooth clear marginsand bright-red glossy floor. Ulcer edges form acushion that smoothly descends to the bottomof the ulcer. In the base of the ulcer there isa dense painless infiltration. Submandibularlymph nodes are enlarged and painless, theskin over them remains unchanged. Thisclinical presentation corresponds with thefollowing disease:

A. Syphilis (initial period)B. Ulcerative cheilitisC. Lip cancerD. Trophic ulcerE. Lupus erythematosus

32. The maxillofacial surgery unit received apatient with complaints of inability to closehis mouth. This condition occurred whenthe patient was biting an apple. Objectivelythere is a frightened expression on thepatient’s face, the mouth is open wide, thechin is displaced to the left, salivation is

Page 12: Krok 2 - Центр тестування при МОЗ УкраїниKrok 2 Stomatology () Терапевтична стоматологiя 2 1. Biopsy material was obtained from the

Хiрургiчна стоматологiя 12

observed. Palpation through the externalacoustic meatus detected no movements ofthe right articular head. What is the mostlikely diagnosis?

A. Right temporomandibular joint dislocationB. Fracture of the mandibular processC. Acute temporomandibular arthritisD. Temporomandibular joint pain dysfunctionsyndromeE. Bilateral temporomandibular jointdislocation

33. A 37-year-old patient has symmetricalface; the mucosa in the area of the 12 toothroot apex projection is pale pink; palpation ispainless; the tooth crown is destroyed by 1/3;percussion is painless. X-ray: the root canal ofthe 12 tooth is filled to the apex; granuloma4 mm in diameter surrounds the root apex.Choose the method of surgical treatment:

A. Granuloma removal with root apexresectionB. Root hemisectionC. Coronary radicular tooth separationD. Root amputationE. Tooth extraction

34. A patient is diagnosed with mandibularameloblastoma. What type of surgery isrecommended for this patient?

A. Mandibular resection at the distance of 1.5cm from the lesion focusB. Tumor curettage within healthy tissueC. Tumor cryodestructionD. CystectomyE. Only conservative treatment

35. A 48-year-old man presents withverruciform, dense, gray-white growths on thebuccal mucosa. The growths protrude abovethe neighbouring tissues and are surroundedby keratinized gray-white spots that cannot bescraped off. Make the provisional diagnosis:

A. Verrucous leukoplakiaB. Erosive leukoplakiaC. Bowen’s diseaseD. Erythroplasia of QueyratE. Papillomatosis

36. During preventive examination apatient was diagnosed with precanceroushyperkeratosis of the lower lip vermillionborder. What treatment should be prescribed?

A. Surgical removal of the focus within healthytissuesB. Surgical removal of the focus within healthytissues + close-focus roentgenotherapyC. No treatment is requiredD. Surgical removal of the focus within healthytissues + chemotherapyE. Palliative treatment

37. A 21-year-old man 3 hours ago receiveda knife wound to the right cheek; the

wound was initially bleeding, but by thetime of examination the bleeding has alreadystopped. Objectively in the area of the rightcheek there was a wound 4x1 cm with evenmargins that does not penetrate into the oralcavity and is filled with clotted blood. Whatsutures should be applied to the wound in thiscase?

A. Primary blind sutureB. Primary apposition sutureC. Primary approximation sutureD. Early secondary sutureE. Late secondary suture

38. A 22-year-old patient has sufferedunilateral linear fracture in the area of thegonial angle. Immobilization was providedwith full dental brace with loops andintermaxillary elastic expansion. Recoverywas uncomplicated. The brace should beremoved after:

A. 3 weeksB. 2 weeksC. 1 weekD. 10 daysE. -

39. A 25-year-old man was brought to thehospital for specialized medical care 48 hoursafter he received a shrapnel wound of themaxillofacial area. Examination shows alarge gaping wound of irregular shape in thesoft facial tissues. Wound edges are infectedand swollen. What type of surgical woundtreatment should be conducted at this stage?

A. Late primaryB. Early primaryC. SecondaryD. Delayed primaryE. -

40. An 18-year-old girl complains of pain inher parotid regions and general indisposition.Examination shows bilateral swelling of theparotid regions, painful mouth opening. Inthe oral cavity, mucosa near the openingsof parotid salivary glands is hyperemicand swollen. There are signs of generalintoxication: body temperature of 38oC andmyalgia. Make the provisional diagnosis:

A. Acute epidemic parotitisB. Acute non-epidemic parotitisC. Phlegmon of the parotid-masseteric regionD. Acute suppurative lymphadenitisE. Herzenberg pseudoparotitis

41. A 29-year-old man is diagnosed withmedial maxillary fracture. The line of thefracture is symmetrical on both sides. In thiscase sensory perception in the teeth andmucosa is likely to be disturbed within thefollowing interval:

Page 13: Krok 2 - Центр тестування при МОЗ УкраїниKrok 2 Stomatology () Терапевтична стоматологiя 2 1. Biopsy material was obtained from the

Хiрургiчна стоматологiя 13

A. From tooth 15 to tooth 25B. From tooth 11 to tooth 21C. From tooth 13 to tooth 23D. From tooth 18 to tooth 28E. Sensitivity of the teeth and mucosa willremain undisturbed

42. A 28-year-old woman complains ofdull pain when biting on tooth 16. Uponexamination she was diagnosed withexacerbation of chronic periodontitis. She isprescribed extaction of tooth 16. What forcepsshould be used for tooth extraction in thiscase?

A. S-shaped right forcepsB. S-shaped left forcepsC. Straight forcepsD. Beak-shaped non-crushing forcepsE. Beak-shaped curved forceps

43. A patient is prescribed mandibularvestibuloplasty. What pathology is likely tooccur if the vestibule of the mouth is tooshallow?

A. Diseases of periodontal tissuesB. Delayed growth of the jawC. Overcrowding of the front teethD. Parafunction of mimic musclesE. Deep occlusion

44. To extract tooth 27, the patient was giventuberal and palatal anesthesia. When theneedle was being pulled out from the softtissues after the palatal anesthesia, the patientdeveloped bleeding. What should be done tostop the bleeding in this case?

A. Press the place of injection with a tamponB. Place suturesC. Intravenous administration of 10% calciumchloride solutionD. Intramuscular administration of vicasol(menadione)E. Ligate the external carotid artery

45. How often should a dentist’s office bethoroughly cleaned?

A. No less than once a weekB. No less than once a monthC. No less than once a yearD. No less than once every 2 weeksE. No less than twice a week

Page 14: Krok 2 - Центр тестування при МОЗ УкраїниKrok 2 Stomatology () Терапевтична стоматологiя 2 1. Biopsy material was obtained from the

Ортопедична стоматологiя 14

1. A removable full denture for the lower jawis being made for a 75-year-old man. Objectivelythe alveolar process is slightly atrophied. Herbsttests are performed during fitting of an impressiontray. When lips are stretched forwards the trayslips off. Where should the tray edge be shortenedin this case?

A. From canine to canine on the vestibular sideB. From canine to canine on the lingual sideC. From behind the mandibular tuberosity to themylohyoid lineD. Along the mylohyoid lineE. In the premolar area on the lingual side

2. A 40-year-old man presents with a medialdefect of the hard palate 2x3 cm in size. Dentitionis intact. What type of obturator would be optimalin this case?

A. Palatal plateB. Plate prosthesis with obturatorC. Pomerantseva-Urbanska obturatorD. Floating obturatorE. Ilina-Markosian obturator

3. After a clasp-retained (bugel) maxillarydenture is made, it is necessary to assess thequality of the newly-made construction. Widthof the denture arch should be:

A. 5-8 mmB. 1.5-2 mmC. 3-5 mmD. 10-12 mmE. 12-15 mm

4. A 32-year-old woman needs a denture. Afterobjective examination the decision was made infavor of porcelain-fused-to-metal crown. Whatmaterial should be used in this case to obtain theimpression?

A. StomaflexB. RepinC. StomalginD. StensE. Orthocor

5. A 60-year-old patient came to the maxillofacialinpatient department. He complains of pain,bleeding, and chewing problems. He has a historyof mandibular trauma at the level of the centralincisors. Objectively the traumatized place isswollen, mouth opening is unrestricted. He wasdiagnosed with medial mandibular fracture.Both jaws are edentulous. What splint would beoptimal in this case?

A. Port gingival splintB. Limberg gingival splintC. Rudko applianceD. Gunning gingival splintE. Vankevych dentogingival splint

6. A 43-year-old woman complains ofmobility and displacement of her upperfront teeth. Objectively: dental formula is

17 16 15 14 13 12 11 21 22 23 24 25 26 2747 46 45 44 43 42 41 31 32 33 34 35 36 37 .

Teeth 12 11 21 22 are slanted towardsthe vestibular side, diastema and tremata areobserved, I-II degree teeth mobility is detected.Select the orthodontic appliance for correction of

teeth misalignment as a part of complex treatmentof periodontal disease:

A. Palatal plate with vestibular archB. Bynin applianceC. Schwartz applianceD. Katz crownE. Palatal plate with inclined plane

7. A 39-year-old woman complains of teethmobility in her lower jaw. Objectively her dentalformula is as follows: 17 16 15 14 13 12 11 / 2122 23 24 25 26 27, 47 46 45 44 43 42 41/31 3233 34 35 36 37. The teeth are intact, crowns aretall. Gingival pockets and mobility of the I andII degrees are observed in teeth 42 41/31 32. Toimmobilize the mobile teeth, a cap splint withfixed crowns was made for 43/33. What type ofstabilization provides this splint?

A. FrontalB. ParasagittalC. Frontal-sagittalD. SagittalE. Circumferential

8. A 35-year-old man suffers from localizedperiodontitis of the front teeth on his lowerjaw. Objectively on the upper jaw his dentitionis uninterrupted, while on the lower jaw teeth48, 47, 46, 45, 35, 36, 37, 38 are missing, otherteeth present with mobility of the II degree,tremata, and diastema. What tactics should aprosthodontist choose in this case?

A. Orthodontic treatment followed by splintingand prosthesis-makingB. Restore the height of the occlusion with apartial removable laminar dentureC. Restore the height of the occlusion with aclasp-retained (bugel) dentureD. Splinting of the mobile teeth followed byprosthesis-makingE. Temporary splinting of the teeth on the lowerjaw

9. What medical establishment provides dentaland prosthodontic (making of splints, appliances,and dentures) treatment for those who receivedmaxillofacial injuries on the battlefield?

A. Specialized surgical field hospitalB. Separate medical battalionC. Separate medical platoonD. Evacuation hospitalE. Civilian medical establishment

10. One of the stages of making a removablefull denture includes fitting of an impression trayand obtaining the functional impression. Whatmaterial is applied to the edges of the impressiontray?

A. OrthocorB. PlasterC. KromopanD. ThiodentE. Stomalgin

11. The 40-year-old woman complains of inabilityto properly masticate due to the loss of thefollowing lateral teeth: 18, 16, 15, 25, 26, 28,38, 35, 36, 44-46, and 48. The rest of herteeth present with the I-II degree of mobility.

Page 15: Krok 2 - Центр тестування при МОЗ УкраїниKrok 2 Stomatology () Терапевтична стоматологiя 2 1. Biopsy material was obtained from the

Ортопедична стоматологiя 15

Generalized periodontitis is observed. Whatdenture construction would be optimal in thiscase?

A. Removable dental splintB. Metal-based dentureC. Removable laminar dentureD. Fixed dental bridgeE. -

12. A 55-year-old patient requires a denture.Objectively: Kennedy’s I class dentition defect;the 16, 17, 18, 26, 27, and 28 teeth are missing.The patient presents with fixed occlusion. The15 and 25 teeth have low crowns with pooranatomic contours, intact. Clasp-retained (bugel)removable partial denture is being made for thepatient. What fixation system would be optimal inthis case?

A. Telescopic fixationB. AttachmentsC. Roach clasp (clammer)D. Aker-Roach combined clasp (clammer)E. Continuous clasp (clammer)

13. A 54-year-old patient is prescribed full castporcelain-fused-to-metal dental bridges for theupper and lower jaws. What type of impressionmaterial should be chosen in this case?

A. SiliconeB. PlasterC. ThermoplasticD. Zinc oxide eugenolE. Alginate

14. A 28-year-old patient complains of aestheticaldefect. Objectively the crowns of 12, 11, 21, 22,and 23 are destroyed by caries by over 2/3 of theirheights. The patient has orthognathic occlusion.X-ray shows the root canals of these teeth to becompletely filled; no pathologic changes detectedin the periapical tissues. What approach to teethrestoration would be optimal in this case?

A. Stump inlays and porcelain-fused-to-metalcrownsB. Plastic-faced stump inlaysC. Crown restoration with photopolymer fillingsD. Combined swaged crowns, made in BorodiuktechniqueE. Immediate denture with posts

15. On objective examination a 59-year-old manwith the edentulous mandible presents with boneprotrusions and mobile areas of the alveolar crest.To ensure proper fixation of the denture andeven load distribution the following functionalimpression should be made:

A. DifferentiatedB. Complete anatomicalC. CompressionD. DecompressionE. Combined

16. A 42-year-old man came to the prosthodonticclinic. He complains of inability to chew his fooddue to partial loss of teeth. Objectively in thelateral regions teeth 18, 16, 15, 25, 26, 28, 38, 35,36, 44, 46, 48 are missing. Other teeth present withI-II degrees of mobility. The patient is diagnosedwith generalized periodontitis of the II degree.

What type of denture would be optimal for thispatient?

A. Clasp-retained (bugel) dentures with splintingelementsB. Metal-based denturesC. Non-removable full cast dental bridgesD. Removable partial laminar dentureE. -

17. An 18-year-old patient with complaint oflarge diastem has made an appointment withprosthodontics specialist. Objectively: there isfull lateral displacement of central incisors dueto absence of the 12th and 22nd teeth. Whatinstrument is the most advisable for moving thecentral incisors closer together?

A. Korkhaus applianceB. Vasylenko applianceC. Simple cotton ligatureD. Kalvelis applianceE. Begg appliance

18. A 25-year-old man complains of incorrectlypositioned maxillary left central incisor due totrauma sustained 2 months ago. Objectively tooth21 is rotated around its axis into palatal position.What would be the most advisable treatmentmethod for correction of this defect?

A. Orthodontic treatmentB. Surgical treatmentC. Instrumental surgical treatmentD. Splinting followed by prosthetic treatmentE. -

19. A 46-year-old man needs a classic 2-stagesurgical installation of implants into the upperjaw. How long should be the interval betweenstages 1 and 2?

A. 3-6 monthsB. 1 monthC. 1.5 monthsD. 2 monthsE. 0.5 months

20. A 47-year-old man has lost his lower leftpremolars. After that the teeth at the edges ofthe defect have been gradually sliding insidethe defect. At the same time the antagonistteeth started to protrude toward the edentuloussegment of the alveolar bone. In the medicalliterature, such clinical presentation is called:

A. Popov-Godon phenomenonB. Castaigne syndromeC. Papillon-Lefevre syndromeD. Kourliandski phenomenonE. Phenomenon of relative tooth-alveolarlengthening

21. A 57-year-old man presents with habitualmandibular dislocation. To reduce mouthopening, Yadrova apparatus was made. How longshould the treatment last in this case?

A. 3 monthsB. 6 monthsC. 9 monthsD. 12 monthsE. 18 months

22. A 27-year-old woman complains of recurrent

Page 16: Krok 2 - Центр тестування при МОЗ УкраїниKrok 2 Stomatology () Терапевтична стоматологiя 2 1. Biopsy material was obtained from the

Ортопедична стоматологiя 16

loss of a tooth filling in the lower right jaw.Objectively: in the 46 tooth on the masticatoryapproximal surface there is a defect of hard toothtissues affecting 1/3 of the tooth crown, no toothdiscoloration; positive, quickly abating reactionto cold stimulus is observed. What dentureconstruction would be optimal in this case?

A. Dental inlayB. Combined crownC. Porcelain-fused-to-metal crownD. Plastic crownE. Partial crown

23. To make the external prosthesis for a 62-year-old man it is necessary to obtain a Hippocratesfacial moulage of this patient. What impressionmaterial should be used?

A. PlasterB. DentafolC. StensD. StomaflexE. Repin

24. A patient needs a removable full laminardenture for the upper jaw. Objectively on themucosa of the denture bed there are numerousdense papillomas of varying size. What tacticsshould the dentist choose?

A. Remove papillomas and make a denture with adouble layered base lined with elastic materialB. Reduce the area of the denture baseC. Make a 3D-model of the denture baseD. Removable dentures are contraindicated in thiscaseE. Make a denture base from metal

25. One week ago a patient received removabledentures for the upper and lower jaws. During afollow-up visit to the dentist the patient complainsof accidental cheek-biting and pain. Objectivelyhe presents with hyperemia, edema, ulceration ofthe buccal mucosa along the line of teeth closurein the area of the artificial molars. What is thelikely cause of this condition?

A. Cusps of the molars on the upper and lower jaware closing in one vertical planeB. Due to diabetes mellitus, mucosa is moresusceptible to traumaC. Toxic-allergic reaction to the plastic componentsof denture baseD. Fixed anterior occlusionE. Incorrectly measured height of the occlusion

26. A 70-year-old man has edentulous maxilla.Objectively maxillary tuberosity and alveolarprocesses are completely atrophied; palatine vaultis flat, its mucosal layer is moderately pliant. Inthis case the patient’s atrophic edentulous maxillacan be classified as:

A. Schroeder class IIIB. Keller class IIIC. Schroeder class IID. Keller class IIE. -

27. In the epicenter of a natural disaster, a manwith a lacerated wound of soft facial tissues wasfound. The wound cuts open the patient’s lowerlip and chin; external bleeding is observed. The

patient is conscious. What first aid should beprovided for this man?

A. Stop the external bleeding, pack thewound, apply sterile bandage, provide transportimmobilizationB. Use a syrette from the personal medicalkit to provide anesthesia with 2% promedol(trimeperidine) solutionC. Provide primary surgical processing of thewound, place the suturesD. Position the flaps correctly and place the U-shaped mattress suturesE. Thoroughly ligate the vessel in the wound,position the flaps correctly, place the U-shapedmattress sutures, apply sterile bandage

28. The medical station of a regiment received apatient with signs of bilateral mandibular fracture.What is the main task of first aid in this case?

A. To control shock, bleeding, and asphyxia and toprovide transport immobilizationB. To check and correct previously appliedbandagesC. To administer analgesics and cardiacmedicationsD. To clean the oral cavity from blood clots, toothshards, and bone fragmentsE. To provide symptomatic therapy and care

29. A removable partial laminar denture for theupper jaw is being made for the patient. Thecentral occlusion is determined and fixed. Whatstage is next?

A. Fitting the wax model of the denture in the oralcavityB. Arrangement of artificial teethC. Replacing wax with plasticD. Determining the edges of the baseE. Measuring the interalveolar height

30. A porcelain-fused-to-metal crown for tooth 11is being made for the patient. The tooth will beleft vital. What measures should be taken duringtreatment to prevent pulpitis in this tooth?

A. Pharmaceutical crownB. Fluorine preparationsC. Calcium preparationsD. Anti-inflammatory preparationsE. Physical therapy

31. A 55-year-old man came to the prosthodonticclinic to have a denture made for him. Tooth 11is missing in the patient. Two days ago he wasreleased from the inpatient unit after a case ofmyocardial infarction. What tactics should thedentist choose?

A. Make a temporary removable dentureB. Make a clasp-retained (bugel) removable partialdentureC. Make a dental bridge with 12 and 21 asabutment teethD. Temporarily refrain from making a dentureE. Perform implantation

32. During his shift a dentist of the prosthodonticsunit has consulted and examined the patient,checked the crowns, installed dental bridges,measured centric jaw relation, and madenecessary corrections to the removable dentures.

Page 17: Krok 2 - Центр тестування при МОЗ УкраїниKrok 2 Stomatology () Терапевтична стоматологiя 2 1. Biopsy material was obtained from the

Ортопедична стоматологiя 17

What is the normal workload of a dentist pershift and per month measured in arbitrary unitsof labor intensity (AU) in a five-day work weekand according to the normatives for provision ofhealthcare to the adults?

A. 6 AU - 126 AUB. 5 AU - 105 AUC. 7 AU - 147 AUD. 8 AU - 168 AUE. 9 AU - 189 AU

33. A non-removable porcelain-fused-to-metaldental bridge is being made for a 35-year-oldwoman. Two-layer impressions were obtainedusing ”Silafex” silicone material. The impressionwere sent for disinfection. What antiseptic shouldbe used to process the impressions?

A. 0.5% hypochlorite sodium solutionB. 3% chloramine solutionC. 70% alcohol solutionD. 1.5% sodium bicarbonate solutionE. 0.5% hydrogen peroxide solution

34. A 29-year-old man needs to replace a numberof dentition defects on his upper jaw. Thedefects can be replaced with dental bridges withaesthetical coating. ”Sinma-M” plastic coating issecurely attached to the frame of the full castmetal-plastic dental bridges with:

A. Retention spheresB. ”Conalor” dental resinC. EDA varnishD. SandblastingE. Thorough polishing of the frame

35. A 60-year-old man needs a removable partialdenture for the upper jaw. Arrangement of theartificial teeth on the wax base was checked in theoral cavity of the patient. What is the next stagein the making of a removable partial denture?

A. Fitting and fixation of the removable partialdentures in the oral cavityB. Measurement and fixation of the centric jawrelationC. Placing the plaster casts in an articulatorD. Final modelling of the denture baseE. Final polishing of the denture

36. A 64-year-old man undergoes fixation of aremovable partial denture for the lower jaw. Thedentist examined the denture base and noticedthere a clearly visible ”marble pattern”. This netof white lines runs through the whole plastic base.What is the most likely cause of this defect?

A. Disturbed process of acrylic resinpolymerizationB. Uneven moistening of acrylic resin with amonomerC. Disturbed process of denture base polishingD. Foreign admixtures in the denture baseE. Fractures in the denture base caused bymechanical damage

37. A 26-year-old woman needs dental prosthetics.Objectively the crown of 16 is destroyed by 1/3. Itsanatomical shape will be restored with a porcelaininlay. Direct method is chosen for inlay-making.What should be used to obtain the impression ofthe formed inlay cavity?

A. Gold or platinum foilB. Silicone materialsC. Thermoplastic materialsD. WaxE. Solid crystal materials

38. A 35-year-old man needs an adhesive dentalbridge. Objectively tooth 15 is missing. Abutmentteeth 14 and 16 are firm. Teeth crowns are ofmedium height. Dental equator is markedlyvisible. The patient has orthognathic occlusion.What will most likely be an abutment part of thisadhesive denture?

A. Whole or perforated cast overlaysB. Full cast crownsC. Combined crownsD. Stump inlaysE. Equator crowns

39. A 45-year-old man complains of painand crepitation in the temporomandibularjoint during the movements of the lower jaw.Objectively: the face is symmetrical, the mouthopens with slight displacement to the left.Dentition is intact. On occlusiography there weredetected centric and eccentric supracontacts.What treatment methods should be applied inthe first place?

A. Selective teeth shavingB. Mouthguard for muscle relaxationC. Appliances that limit mouth openingD. Mouthguards that increase the height of centralocclusionE. Lower jaw immobilization

40. A 42-year-old woman after examinationwas diagnosed with closed medial mandibularfracture in the area of 31, 41, non-displaced.Crown portions of her other teeth are intact.Additionally she was diagnosed with chronicgeneralized parodontitis of moderate severity.What type of splint would be the most advisablefor fracture treatment in this patient?

A. Weber dentogingival splintB. Tigerstedt splintC. Tigerstedt splint with anchor hooksD. Vankevych dentogingival splintE. Port gingival splint

41. A 37-year-old patient complains of anaesthetic defect. Objectively: the 13 tooth isdestroyed by 2/3. The tooth is pulpless, the rootcanal is filled. How deep should the root canal beopened for pivot crown in this patient?

A. 2/3 of the root canalB. 1/3 of the root canalC. 3/4 of the root canalD. 1/2 of the root canalE. Full length of the root canal

42. A 46-year-old man, a teacher, complains oflower teeth mobility that impedes the process ofbiting. Objectively the dentition is uninterrupted,front teeth demonstrate the II degree of mobility.X-ray shows straight and filled root canals of 32,31, 41, and 42. What appliance will stabilize frontteeth while retaining their aesthetic appearance?

Page 18: Krok 2 - Центр тестування при МОЗ УкраїниKrok 2 Stomatology () Терапевтична стоматологiя 2 1. Biopsy material was obtained from the

Ортопедична стоматологiя 18

A. Mamlok’s dental splintB. Removable segmented splint for the front teethC. Cap splintD. Splint with embrasure claspsE. Semicrown splint

43. A 40-year-old patient complains of pain inthe tragus area, clicking during mouth opening,stuffed ears. Objectively: the face is symmetrical,mouth opening path is straight. Dentition defectcan be estimated as the I class by Kennedy; the18, 17, 16, 26, 27, 28 teeth are absent. In this casethe load would be the most traumatizing for thefollowing anatomical structure:

A. Interarticular diskB. Articular capsuleC. Articular headD. Distal slope of the articular tubercleE. Socket floor of the temporal bone

44. A 57-year-old man complains of pain andcreaking in his right temporomandibular jointwhen eating. In the evening the signs diminish.The pain is observed for the last 2-3 years.Objectively the face is symmetrical, mouthopening is reduced to 2.5 cm. Molars are missingon both jaws. X-ray shows deformed articularsurfaces of the right temporomandibular joint.What is the most likely diagnosis?

A. Arthrosis of the right temporomandibular jointB. Ankylosis of the right temporomandibular jointC. Acute arthritis of the right temporomandibularjointD. Pain dysfunction syndrome of the righttemporomandibular jointE. Contracture of the right temporomandibularjoint

45. A 44-year-old man came to the dentalpolyclinic to have a denture made for him.He complains of problematic chewing and

aesthetical defects. Objectively teeth 14, 15, and22 are missing. According to Agapov, the loss ofmasticatory efficiency in this case is:

A. 18%B. 16%C. 9%D. 20%E. 12%

46. A 21-year-old man complains of difficultmouth opening and visible facial deformationthroughout the last 2 years. He has a historyof facial trauma several years ago. Examinationshows the right side of the face to be flattened,the chin is displaced to the right. Palpation detectsdiminished excursion of the articular head in theleft lower jaw. Mouth opening is reduced to 1 cm.Make the preliminary diagnosis:

A. Ankylosis of the left temporomandibular jointB. Exacerbated chronic arthritis of the lefttemporomandibular jointC. Acute arthritis of the left temporomandibularjointD. Anterior dislocation of the lower jawE. Arthrosis of the left temporomandibular joint

47. A 50-year-old woman complains of painand creaking in her temporomandibular joint.Objectively there are multiple premature occlusalcontacts and localized pathologic tooth wear.The teeth are stable, with exposed cervices; thegingival margin is hyperemic. Selective teethshaving is planned. How many visits to the dentistare necessary for the full procedure?

A. 3-4 visits at one-week intervalsB. 2-3 visits at one-month intervalsC. 1-2 visits within a yearD. A single visitE. 2-3 visits at one-day intervals

Page 19: Krok 2 - Центр тестування при МОЗ УкраїниKrok 2 Stomatology () Терапевтична стоматологiя 2 1. Biopsy material was obtained from the

Дитяча терапевтична стоматологiя 19

1. A 18-year-old young man complainsof bleeding and pain in his gums. Thedisease onset was 4 days ago. Objectivelythe skin is pale, body temperature is 38.5oC .Submandibular lymph nodes on the leftare enlarged, painful, non-fused with thesurrounding tissues. The gingival papillae andgingival margin in the area of 33, 34, 35, 36,and 37 are hyperemic, ulcerated, and coveredin necrotic deposit. Teeth are covered withsoft dental plaque. Make the diagnosis:

A. Necrotizing ulcerative gingivitisB. Acute catarrhal gingivitisC. Chronic hypertrophic gingivitisD. Chronic catarrhal gingivitisE. -

2. A 13.5-year-old girl complains of swollengums that bleed when she brushes her teeth.This condition has been observed for thelast half a year. Objectively gingival mucosasurrounding the front teeth of the upper jawis swollen and cyanotic. Interdental papillaeare round, dense, and enlarged, they coverthe crowns by 1/3 of their height. Teeth 13and 23 are positioned vestibularly. Make theprovisional diagnosis:

A. Chronic hypertrophic gingivitisB. Chronic catarrhal gingivitisC. Acute catarrhal gingivitisD. Generalized periodontitis, degree IE. Localized periodontitis, degree I

3. Parents of a 5-year-old girl brought her tothe pediatric dentist for oral cavity sanation.After clinical examination she was diagnosedwith chronic deep caries in 54. What fillingmaterial would be optimal for the treatmentof tooth 54 in this case?

A. Glass ionomer cementB. Zinc phosphate cementC. Silicate cementD. CompositeE. Silicophosphate cement

4. A 7.5-year-old practically healthy childcomplains of crown fracture and pain inthe upper right incisor. Objectively 2/3 ofcrown of 11 is absent, the pulp is exposedand red; on probing it is acutely painful andbleeding; tooth percussion is painful. Thetrauma occurred 2 hours ago. What wouldbe the optimal treatment method in this case?

A. Vital amputationB. Devital amputationC. Vital extirpationD. Devital extirpationE. Biological approach

5. Parents of a 2.5-year-old child complain ofgradual destruction of the upper front teethof their child for the last several months.Objectively there are carious cavities withinmantle dentin on the contact and vestibular

surfaces of 52, 51, 61, and 62. The cavitiesare filled with softened pigmented dentin thatcan be easily removed with dental excavator.Make the provisional diagnosis:

A. Acute median cariesB. Chronic deep cariesC. Acute deep cariesD. Chronic median cariesE. Chronic superficial caries

6. A 6-month-old child was diagnosed withbilateral bronchopneumonia and prescribedbroad-spectrum antibiotics. The child isformula-fed and presents with maldigestion(dyspepsia). What pathologic changes canoccur in the hard dental tissues in this case?

A. Systemic enamel hypoplasiaB. Hutchinson teethC. Local hypoplasiaD. Stainton-Capdepont syndromeE. Fournier teeth

7. A 6-year-old girl took paracetamol to treata case of URTI two days ago, which resultedin the development of her present condition.The disease onset was acute with temperatureincrease up to 39.8oC . Objectively thereare cockade-shaped maculopapular rasheson her face. The vermillion border isswollen, hyperemic, covered in massivebrown crusts, and presents with bleedingcracks. Conjunctivitis is detected. Swollenand hyperemic oral mucosa presents withnumerous erosions covered with fibrinousincrustations; the erosions are sharply painfulon palpation. What is the most likelydiagnosis?

A. Stevens-Johnson syndromeB. Erythema multiforme exudativumC. Acute herpetic stomatitisD. Chronic recurrent aphthous stomatitisE. Pemphigus

8. Decay-missing-filled index of a 6.5-year-old child is DMF+df=7. Fedorov-Volodkinahygiene index is 1.5. Fissures in the permanentmolars are intact, deep, and open. Whatmethod of caries prevention would beadvisable in this case?

A. Noninvasive fissure sealingB. Invasive fissure sealingC. Application of remineralization solutionD. Application of fluoride varnishE. Electrophoresis-aided application ofremineralization solution

9. A 7-year-old boy was brought to thedentist. Objectively his gums are hyperemicand bleeding, the teeth are mobile andcovered in plaque, their roots are exposed,periodontal pockets are pathologic and filledwith granulations. The child is registeredfor regular check-ups with the pediatrician.According to the mother, lately the child’s

Page 20: Krok 2 - Центр тестування при МОЗ УкраїниKrok 2 Stomatology () Терапевтична стоматологiя 2 1. Biopsy material was obtained from the

Дитяча терапевтична стоматологiя 20

condition has been deteriorating. Theboy presents with diabetes insipidus andexophthalmos. What provisional diagnosiscan be made?

A. Hand-Schuller-Christian diseaseB. Taratinov disease (eosinophilic granulomaof bone)C. Letterer-Siwe diseaseD. Gaucher diseaseE. Papillon-Lefevre syndrome

10. Parents of a 5-year-old child came to thedentist for sanation of the child’s oral cavity.Objectively on the masticatory surfaces ofteeth 54, 64, 74, 75, and 85 there are cariouscavities within the softened mantle dentin.The child is anxious and misbehaves. Whattreatment tactics would be optimal in thiscase?

A. ART techniqueB. Impregnation treatmentC. Deep fluoridationD. Preventive fillingE. Invasive sealing

11. A 10-year-old child complains of pain,dryness, enlargement, and peeling of thelips. Exaination detected hyperemic labialmucosa and enlarged lips; when the lips arepulled taut, secretion resembling ”dewdrops”is being produced from minor salivary glands.What is the most likely provisional diagnosis?

A. Cheilitis glandularisB. Meteorological cheilitisC. Actinic cheilitisD. Exfoliative cheilitisE. Allergic contact cheilitis

12. An 8-year-old girl complains of bleedinggums. The child has diabetes mellitus.Objectively the interdental papillae betweenthe teeth of the upper and lower jaw arehyperemic, swollen, and cover the crowns by1/3 of their height. The teeth exhibit mobilityof the I degree. What investigation techniqueis necessary for diagnosis-making in this case?

A. X-rayB. RheoparodontographyC. Vacuum testD. PMA indexE. Stomatoscopy

13. A 12.5-year-old child came to the dentistfor oral cavity sanitation. Objectively cariesprogression in this child is severe: decay-missing-filled index is DMF=2+0+4=6. Green-Vermillion index is 1.7. Gingival mucosa is palepink. What toothpastes should be prescribedin this case for oral hygiene?

A. Amine fluorideB. EnzymeC. AntisepticD. Salt supplementE. Herbal extract

14. A 6-year-old child for a 3rd day presentswith body temperature up to 38-39oC andpainful swallowing. Objectively on themucosa of the palatine arches, tonsils anda part of the soft palate there are severalerosions up to 2 mm in diameter. The erosionsare situated on the hyperemic mucosa andcovered in whitish plaque. Submandibularlymph nodes are enlarged and painless onpalpation. What is the most likely diagnosis?

A. Herpetic anginaB. DiphtheriaC. Acute herpetic stomatitisD. Infectious mononucleosisE. Erythema multiforme exudativum

15. An 8-year-old boy complains of a dull painin his lower right tooth. The pain appears inresponse to thermal stimuli and slowly abatesafter the stimuli are removed. Objectively onthe approximal surface of tooth 46 there is adeep carious cavity. During necrectomy witha dental excavator, communication betweenthe carious and dental cavities is detected.Probing of the communication area resultsin sharp pain and bleeding. What methodof pulpitis treatment is recommended in thiscase?

A. Vital amputationB. Vital extirpationC. Devital amputationD. Devital extirpationE. Biological approach

16. A 12-year-old boy is in the dentaloffice to complete the treatment of chronicperiodontitis exacerbation in the 36 tooth.There are no complaints. Objectively:occlusive dressing of the 36 tooth is intact;tooth percussion is painless; there are nopathologies of the mucosa surrounding the36; palpation is painless. What material is themost advisable for root canal filling in thiscase?

A. Sealer with gutta-percha postB. Zinc oxide eugenol pasteC. Resorcinol-formalin pasteD. Calcium-containing pasteE. Phosphate cement

17. During planned sanation of a 3-year-oldchild, a deep carious cavity filled with fooddebris and softened dentin is detected intooth 54. After tooth preparation the pulp of54 is bleeding, probing is sharply painful. Totreat pulpitis by means of devital amputation,during the first visit it is necessary to use:

Page 21: Krok 2 - Центр тестування при МОЗ УкраїниKrok 2 Stomatology () Терапевтична стоматологiя 2 1. Biopsy material was obtained from the

Дитяча терапевтична стоматологiя 21

A. Paraformaldehyde pasteB. Zinc-eugenol pasteC. Dentin pasteD. Calcium-containing pasteE. Iodoform paste

18. A 12-year-old girl complains ofspontaneous pain in tooth 16 with shortperiods of remission (10-20 minutes) thatdeveloped 4 days ago. The pain increasesat night and irradiates to the upper rightjaw. Examination revealed a deep cariouscavity in 16, the dentin is soft, the probingis painful at all points of the cavity floor,vertical percussion is slightly tender, the painincreases in response to thermal stimuli. EPTis 25 microamperes. Make the diagnosis:

A. Acute diffuse pulpitisB. Chronic concrementous pulpitisC. Acute apical periodontitisD. Acute local pulpitisE. Chronic fibrous pulpitis

19. During preventive examination a 10-year-old child was found to have matt white spotson the vestibular surface of the 11 and 21teeth. The spots are located in the area oftooth cervix. Enamel surface in the affectedarea is undamaged and can be stained withmethylene blue. Subjectively the child hasno complaints. What is the optimal treatmentmethod in this case?

A. Remineralization therapyB. Preparation and fillingC. Impregnation methodD. ART techniqueE. Teeth sealing

20. A 14-year-old boy complains of rapidwearing-off of tooth crowns. Objectively:tooth crowns are worn-off by 1/3. Enameleasily chips off and is pale gray in color. Makethe diagnosis:

A. Stainton-Capdepont syndromeB. Dentinogenesis imperfectaC. FluorosisD. Systemic hypoplasiaE. Focal hypoplasia

21. Objective examination of a 10-year-oldchild revealed slight hyperemia, infiltration,and dryness of the whole surface of thevermillion border. Architectonics of the lips isdisturbed. Dryness and contracted sensationare observed in the lips, especially during coldseasons. Make the provisional diagnosis:

A. Meteorological cheilitisB. Atopic cheilitisC. Allergic contact cheilitisD. Exfoliative cheilitisE. Cheilitis of microbial origin

22. A 2.5-year-old child has fever up to38.5oC , low appetite, rashes in the oral cavity.The disease onset was 3 days ago. Objectively:

the skin of the perioral area is covered inscarce vesicles with clear content. Within theoral cavity on the buccal and lingual mucosathere are sharply painful erosions, 2-3 mmin size, with white coating and hyperemiccrown. The gums are swollen, hyperemic. Thesubmandibular lymph nodes are enlarged,painful on palpation. Make the diagnosis:

A. Acute herpetic stomatitisB. Stevens-Johnson syndromeC. Erythema multiforme exudativumD. Stomatitis with the background of infectiousmononucleosisE. Stomatitis with the background ofchickenpox

23. A 10-year-old boy complains of acute painattacks in the area of his upper left teeth.The toothache persisted for a night. Objectiveexamination revealed a carious cavity on themasticatory surface of the 26 tooth withinparapulpar dentin. Probing is sharply painfulat all points of the cavity floor. Markedlypositive reaction to cold water stimulus isobserved. Select the most likely diagnosis:

A. Acute diffuse pulpitisB. Acute serous periodontitisC. Acute suppurative pulpitisD. Acute suppurative periodontitisE. -

24. A 15-year-old girl complains of brief painattacks in her teeth due to chemical stimuli.Objectively: on the contact surfaces of the11, 21, 22 teeth there are enamel areas mattwhite in color, with lost shine, covered in largeamount of dental deposit. Enamel is softenedand can be easily chipped off with excavator.Probing of lesions is painless. Percussion ispainless. No reaction to cold stimuli. Makethe diagnosis:

A. Acute superficial cariesB. Acute median cariesC. Acute initial cariesD. Chronic initial cariesE. Chronic superficial caries

25. A 12-year-old boy complains of constantpain in the upper tooth. The pain aggravateson biting. Objectively in 26 there is a deepcarious cavity non-communicating with thedental cavity. Thermal stimuli and probingof the cavity floor are painless. Percussion issharply painful. Mucosa in the area of tooth26 exhibits mild signs of inflammation. X-ray of tooth 26 shows no changes. Make thediagnosis:

A. Acute serous periodontitisB. Acute suppurative periodontitisC. Exacerbation of chronic periodontitisD. Acute odontogenic periostitisE. Acute suppurative pulpitis

26. A 16-year-old girl complains of constant

Page 22: Krok 2 - Центр тестування при МОЗ УкраїниKrok 2 Stomatology () Терапевтична стоматологiя 2 1. Biopsy material was obtained from the

Дитяча терапевтична стоматологiя 22

dull pain in her lower tooth. The painaggravates on biting. Objectively in 46 thereis a deep carious cavity communicating withthe dental cavity. Comparative percussion issharply painful. Thermal stimuli and probingare painless. Mucosa in the area of tooth 46is markedly hyperemic, swollen, painful onpalpation. X-ray shows an irregularly shapedfocus of bone tissue destruction with blurredmargins. Make the diagnosis:

A. Exacerbation of chronic granulatingperiodontitisB. Acute serous periodontitisC. Acute suppurative periodontitisD. Exacerbation of chronic granulomatousperiodontitisE. Exacerbation of chronic fibrousperiodontitis

Page 23: Krok 2 - Центр тестування при МОЗ УкраїниKrok 2 Stomatology () Терапевтична стоматологiя 2 1. Biopsy material was obtained from the

Дитяча хiрургiчна стоматологiя 23

1. A 10-year-old child is referred bythe orthodontist for extraction of tooth 53.Objectively the crown of 53 is retained, thetooth is immobile. X-ray of tooth 53 showsroot resorption by less than 1/3. Choose thebest instrument for extraction of tooth 53:

A. Straight crown forcepsB. Straight elevatorC. Root bayonet forcepsD. Crown forceps with S-shaped handlesE. Beak-shaped root forceps

2. An adolescent complains of reduced andpainful mouth opening, difficulties wheneating, and swelling in the left mandibularangle that developed after tooth 37 wasextracted 3 days ago. Objectively the faceis asymmetric due to soft tissue swelling inthe area of the left mandibular angle. Mouthopening is painful and reduced to 2.0 cm.Disturbed occlusion is observed. Palpation ofthe left mandibular angle is painful, the tissuesare soft, bone crepitus is detected. ”Indirectload to the chin” symptom is positive in thearea of the left mandibular angle. The socketof the extracted tooth is packed with iodoformgauze. What is the most likely diagnosis?

A. Left mandibular angle fractureB. Mandibular alveolar fractureC. Anterior mandibular fractureD. Mandibular periostitis on the leftE. Odontogenic mandibular osteomyelitis

3. A 6-year-old child was referred forextraction of the temporary lower centralincisors due to changes in the occlusion. Theteeth exhibit the III degree of mobility. Whattype of anesthesia would be optimal in thiscase?

A. Application anesthesiaB. Conduction anesthesiaC. Inhalation narcosisD. Infiltration anesthesiaE. Intraosseous anesthesia

4. An 8-year-old child was diagnosed withacute odontogenic mandibular periostitis onthe right originating from tooth 74. Whattreatment method would be optimal in thiscase?

A. Extraction of 74, periosteotomy,pharmacotherapyB. Endodontic treatment of 74,pharmacotherapyC. Endodontic treatment of 74, periosteotomyD. Periosteotomy, pharmacotherapyE. -

5. A 7-year-old child complains of pain andswelling in the left submandibular region.The swelling in this region developed 2 daysago. Objectively: the child is in a satisfactorycondition, body temperature is of 37.3oC .The face is asymmetrical due to the soft

tissue swelling in the left submandibularregion. Palpation reveals a round formation2x2 cm in size. The formation is mobile,painful, unattached to the skin. The 74 tooth isdiscolored, percussion is painful. What is theprovisional diagnosis?

A. Acute serous odontogenic lymphadenitis ofthe left submandibular regionB. Acute serous nonodontogeniclymphadenitis of the left submandibularregionC. Acute suppurative odontogeniclymphadenitis of the left submandibularregionD. Phlegmonous adenitis of the rightsubmandibular regionE. Lateral cervical cyst

6. A hit to the face has forced the uppercentral incisors of a 15-year-old boy to sinkinto the jaw to the half of their crown height.What treatment tactics should the dentistchoose?

A. Surgical repositioning of 11 and 21, teethimmobilization, endodontic treatment ifnecessaryB. Extraction of 11 and 21, their replacementwith denturesC. Dynamic observation, endodontic treatmentof 11 and 21 if necessaryD. Transplantation of 11 and 21 into thedentition, endodontic treatmentE. Replantation of 11 and 21 into the dentition,ligation

7. A 7-year-old girl complains of a neoplasmon her lower lip. The neoplasm appeared3 months ago and has been slowly growingsince then. Objectively on the mucosa of thelower right lip there is a round neoplasm0.5-1 cm in diameter. It protrudes from themucosa, has smooth surface, and its clearbluish content is visible through its walls. Onpalpation it is painless and elastic. Make theclinical diagnosis:

A. Retention cyst of the lower lipB. Lymphangioma of the lower lipC. Fibroma of the lower lipD. Hemangioma of the lower lipE. Papilloma of the lower lip

8. An 11-year-old girl was diagnosed withchronic parenchymatous parotitis. What X-ray sign indicates this disease?

A. ”Bunch of grapes” signB. ”Leafless tree” signC. ”String of pearls” signD. Fluctuation signE. Filling defect

9. A 12-year-old boy, due to trauma of the44th and 45th teeth area, suffers from thefollowing: pathologic displacement of themandibular alveolar process; rupture of the

Page 24: Krok 2 - Центр тестування при МОЗ УкраїниKrok 2 Stomatology () Терапевтична стоматологiя 2 1. Biopsy material was obtained from the

Дитяча хiрургiчна стоматологiя 24

alveolar process mucosa. What additionalexamination is necessary to specify thediagnosis?

A. X-ray radiography of mandibula in frontaland lateral projectionsB. X-ray radiography of cranium in axillaryprojectionC. X-ray radiography of mandibula in frontaland Parma projectionD. Tomography of mandibulaE. -

10. During examination the child presentswith micrognathia and open bite. X-ray showsno joint space, the right branch of the lowerjaw immediately continues as the temporalbone. Make the diagnosis:

A. Right-sided bony ankylosis of thetemporomandibular jointB. First and second branchial arch syndromeC. Right-sided sclerosing arthrosis of thetemporomandibular jointD. Right-sided fibrous ankylosis of thetemporomandibular jointE. Chronic right-sided arthritis of thetemporomandibular joint

11. During examination the child presentswith retracted mucosa on the soft palateand uvula. The child was diagnosed withcongenital submucous cleft soft palate. Whatsurgical operation is necessary in this case?

A. VeloplastyB. UranoplastyC. CheiloplastyD. UranostaphyloplastyE. Rhinocheiloplasty

12. A 6-year-old child complains of painand edema in the upper right jaw, bodytemperature up to 37.9oC , and deteriorationof general well-being. Symptom onset was 3days ago. Objectively the face is asymmetricdue to soft tissue edema of buccal andinfraorbital regions on the right. The crown of54 is destroyed by 1/2, percussion is painful;the tooth previously had been treated forcomplicated caries. On the palatine side ofthe affected tooth area there is a painfulinfiltration with fluctuation in its center; thetissues over the infiltration are hyperemic.Make the provisional diagnosis:

A. Acute suppurative periostitis of the maxillaoriginating from tooth 54B. Acute serous periostitis of the maxillaoriginating from tooth 54C. Acute odontogenic osteomyelitis of themaxillaD. Exacerbation of chronic periodontitis of 54E. Chronic odontogenic osteomyelitis of themaxilla

13. A 21-year-old man with facial furunclewas brought to the maxillofacial department.What facial localization of furunclesand carbuncles is often complicated bytrombophlebitis of the angular vein?

A. Upper lip and infraorbital areaB. Lower lip and jaw angleC. Lower lip and chinD. Cheek and parotid-masseteric regionE. Nose and external canthus of the eye

Page 25: Krok 2 - Центр тестування при МОЗ УкраїниKrok 2 Stomatology () Терапевтична стоматологiя 2 1. Biopsy material was obtained from the

Ортодонтiя 25

1. A 10-year-old girl complains of anaesthetic flaw. The anamnesis states, that shehad been sucking her right thumb up to theage of 7. Objectively: the face lower thirdis somewhat reduced. The sagittal fissurebetween the upper and lower incisors, is 9mm wide, class 2 according to the Angleclassification. Eshler-Bittner test leads toinitial temporary improvement of the girl’sface, followed by renewed deterioration.What clinical malocclusion is the most likelyin this case?

A. Maxillary macrognathia and mandibularmicrognathiaB. Maxillary macrognathiaC. Mandibular micrognathiaD. Maxillary prognathism with lateralcompressionE. Mandibular retrognathia

2. During examination of schoolchildren,the orthodontist noticed that some of thempresent with strained orbicularis oris muscle.What dental appliance can be used forexercising of the orbicularis oris muscle?

A. Dass activator applianceB. Frankel functional regulator IIIC. Andresen-Haupl activator applianceD. Angle applianceE. Bruckl appliance

3. After adenotonsillectomia it is necessaryto break the mouth breathing habit in a 4-year-old child. The orthodontist recommendsapplication of an oral vestibular shield(Kerbitz’ vestibular plate). Vestibular shieldfacilitates training of the following muscle:

A. Orbicular muscleB. Temporal muscleC. Masseter muscleD. Lateral pterygoid muscleE. Medial pterygoid muscle

4. A child is 8 years old. There are complaintsof overcrowded upper incisors. Objectively:the first molars closure is of Angle’s I class,frontal overbite is orthognathic. The 12 and 22teeth erupt palatinally with space deficiencyof 2/3 of the tooth crown. The 11 and 21teeth are 10 mm each in cross-section. Thechild has inherited father’s facial type withprognathism and macrodontia of the centralincisors. Choose the preventive treatment,considering this hereditary pathology:

A. Hotz serial extraction to reduce the dentalarchB. Jaw expansion to provide the space for the12 and 21 teethC. Massage of the 12 and 21 teeth area tostimulate their eruptionD. Extraction of the 12 and 21 teeth to reducethe dental archE. Filing down of the 11 and 21 approximalsurfaces to provide the space for the 12 and 22teeth

5. What shape does the upper dental archhave in the permanent dentition?

A. Semi-ellipseB. SemicircleC. ParabolaD. Saddle-shapedE. Trapezoid

6. What prosthodontic appliance hasmechanical type of action?

A. Expansion plate with Coffin springB. Schonherr vestibular plateC. Case obturatorD. Frankel functional regulatorE. Bynin appliance

7. What period of bite formation in a child canbe normally characterized by physiologicaltremata and diastemata?

A. Preparation for the change of dentition(4.5-6 years)B. Formation of milk occlusion (6 months - 3years)C. Changing occlusionD. Permanent occlusionE. It is always a sign of pathology

8. During preventive examination a 5-year-old child was found to have insufficientphysiological attrition of cusps of thedeciduous canines. What treatment tacticsshould a doctor choose?

A. To file down the retained canine cuspsB. Medical examination once a month untilthe incisors are replacedC. Medical examination every 6 months untilthe incisors are replacedD. Medical examination every 6 months untilthe canines are replacedE. No medical intervention is necessary

9. The parents of a 3-year-old child cameto the orthodontist. They complain of anaesthetical defect in the teeth of their child.The child has a history of tongue suckinghabit. In the front region there is a verticalfissure 5 mm. What orthodontic appliance isindicated for the treatment of this pathology?

Page 26: Krok 2 - Центр тестування при МОЗ УкраїниKrok 2 Stomatology () Терапевтична стоматологiя 2 1. Biopsy material was obtained from the

Ортодонтiя 26

A. Kraus plateB. Friel plateC. Schonherr plateD. Kerbitz plateE. Dass activator appliance

10. A 7-year-old boy during primaryexamination was found to have a habit ofsleeping with his fist under the cheek. Whatbite anomaly can develop due to this habit?

A. CrossbiteB. Deep biteC. Mesial biteD. PrognathismE. Open bite

11. To clarify the diagnosis, the orthodontistperformed Eschler-Bittner clinical diagnostictest. As the result the facial signs becamemore marked. What jaw abnormality is themost likely in this case?

A. Maxillary macrognathiaB. Mandibular micrognathiaC. Mandibular macrognathiaD. Maxillary micrognathiaE. Combined pathology of both jaws

Page 27: Krok 2 - Центр тестування при МОЗ УкраїниKrok 2 Stomatology () Терапевтична стоматологiя 2 1. Biopsy material was obtained from the

Ортодонтiя 27

1. A 56-year-old patient suffering fromexacerbation of schizophrenia has beenhospitalised in an oral in-patient departmentwith a diagnosis of the lower jaw displacedfracture in the area of the 34-35 teeth. Whatmethod of treatment should be prescribed?

A. OsteosynthesisB. One arch flat occlusal splintC. Dual splintD. Weber’s dental splintE. Vankevych dental splint

2. A 43-year-old man came to the admissionroom. He complains of facial edema andenlarged upper lip and tongue. This conditiondeveloped approximately 40 minutes ago,when he took aspirin. Objectively the patientis anxious, his skin is of normal color,respiration is partially disturbed. The upperlip, eyelids, cheeks, tongue, and soft palateare swollen. What is the most likely diagnosis?

A. Quincke edemaB. Melkersson-Rosenthal syndromeC. Lymphedema of the lipsD. Allergic contact cheilitisE. Cheilitis glandularis

3. A 45-year-old man came to theprosthodontic department to have a denturemade for him. Before the preparation of thehard tissues of the intact teeth, the patientwas given an infiltration anesthesia with 0.5%lidocaine solution. Several minutes later hedeveloped depressed level of consciousness,his blood pressure dropped, convulsionsstarted, and involuntary urination occurred.What emergency condition did the patientdevelop?

A. Anaphylactic shockB. SyncopeC. Pain shockD. Heart failureE. Collapse

4. During intraoral examination the dentistsuspected that the patient has syphilis. Whatshould the dentist do in this case?

A. Complete the examination and refer thepatient for necessary testsB. Continue the examination and start theteeth preparationC. Inform the patient of the suspecteddiagnosis and end the visitD. Decline to provide dental services for thispatientE. Obtain the impressions to study thediagnostic models

5. A 65-year-old man with fully edentulousupper jaw needs removable full laminardenture. To make the denture, anatomicalimpressions of the upper and lower jaws arenecessary. The patient exhibits heightened gagreflex. What drug can be used to suppress the

gag reflex in this patient?

A. 10% lidocaine solutionB. 10% glucose solutionC. 0.1% atropin solutionD. 1% dimedrol (diphenhydramine) solutionE. 0.2% noradrenaline solution

6. During administration of ultracain solutionfor infiltration anesthesia the condition ofa 22-year-old patient sharply deteriorated.The patient became inert, silent, developedcough attacks, expiratory dyspnea, wheezing.Viscous slimy sputum is being expectorated.What urgent condition did the patientdevelop?

A. Bronchial asthma attackB. Anaphylactic shockC. Hypertensic crisisD. AsphyxiaE. Collapse

7. A 45-year-old patient after administrationof local anaesthesia in preparation for oralsurgery has suddenly felt unwell, developedincreasing edema of laryngeal mucosa andrespiration disorder. The dentist stopped themanipulations in the oral cavity. What type ofasphyxia developed in the patient?

A. StenoticB. DislocationalC. ValvularD. ObturativeE. Aspiration

8. A victim of a traffic accident was deliveredinto the admission room. The patient issupine and unconscious. His skin is cyanotic,respiration is extremely labored, mucosa ispale, blood clots are accumulated in the oralcavity. The patient is diagnosed with displacedbilateral mandibular fracture. What measuresshould be taken to prevent complications inthis case?

A. Fixation of the lower jaw and tongue withstandard Entin’s head-chin strapB. Fixation of the tongue to the patient’s collarC. Tracheostomy and artificial pulmonaryventilationD. Removal of foreign bodies from the oralcavityE. Excision of injured mucosal flaps

9. In the maxillofacial department a 6-month-old boy was hospitalized due tobleeding from a wound in the frenulum ofhis tongue. He has a medical history offrenulotomy the day before. The bleedingstarted 6 hours after the surgery. Suturing thewound was ineffective against the bleeding.The child was brought to the hematologydepartment, where the bleeding was stoppedby intravenous administration of coagulationfactor IX concentrate. Make the diagnosis:

Page 28: Krok 2 - Центр тестування при МОЗ УкраїниKrok 2 Stomatology () Терапевтична стоматологiя 2 1. Biopsy material was obtained from the

Ортодонтiя 28

A. Hemophilia BB. Suppurated tongue woundC. Willebrand diseaseD. Iron-deficiency anemiaE. Idiopathic thrombocytopenic purpura

10. After a tooth extraction, the patientdeveloped a hemorrhage from the socket ofthe extracted tooth. What tactics should thedentist choose in this case?

A. Tightly pack the tooth socket with a strip ofiodoform gauzeB. Place sutures into the soft tissuesC. Insert a gauze pack soaked in 3% hydrogenperoxide solutionD. Administer vicasol (menadione) solutionintramuscularlyE. Administer 5-10 mL of 10% calciumchloride solution intravenously

11. A 22-year-old man is referred forextraction of tooth 36. In the office of thedental surgeon before the anestehesia wasgiven to him he felt weakness and developedtinnitus and visual blackout. Objectivelythe face is pale and covered in cold sweat,blood pressure is 110/70 mm Hg. For severalseconds the patient was unconscious. Whatcomplication occurred during the visit to thedentist?

A. Fainting (Syncope)B. Epileptic seizureC. Anaphylactic shockD. Angina pectorisE. Collapse